A question about limsup of the sequence of the average

The name of the pictureThe name of the pictureThe name of the pictureClash Royale CLAN TAG#URR8PPP











up vote
1
down vote

favorite












Let $a_n$ be a sequence of real non-negative numbers



Define $S_n = fracsum_i=1^n a_in$



Prove that



$$liminf(a_n) leq liminf(S_n) leq limsup(S_n) leq limsup(a_n)$$



I wanted to show that $infa_n:n geq t leq infS_n:ngeq t$ for all $t$



However this doesn't seem to be true



The inequality seems to hold only at n goes to infinity but not every n individually



Can anyone help me?










share|cite|improve this question























  • To see why your proposed inequality is false, you may take the first term to be strictly smaller than the rest of the sequence.
    – GNUSupporter 8964民主女神 地下教會
    4 hours ago










  • @GNUSupporter 8964民主女神 地下教會 Yes that is what I found and therefore I'm stuck.
    – Fluffy Skye
    4 hours ago














up vote
1
down vote

favorite












Let $a_n$ be a sequence of real non-negative numbers



Define $S_n = fracsum_i=1^n a_in$



Prove that



$$liminf(a_n) leq liminf(S_n) leq limsup(S_n) leq limsup(a_n)$$



I wanted to show that $infa_n:n geq t leq infS_n:ngeq t$ for all $t$



However this doesn't seem to be true



The inequality seems to hold only at n goes to infinity but not every n individually



Can anyone help me?










share|cite|improve this question























  • To see why your proposed inequality is false, you may take the first term to be strictly smaller than the rest of the sequence.
    – GNUSupporter 8964民主女神 地下教會
    4 hours ago










  • @GNUSupporter 8964民主女神 地下教會 Yes that is what I found and therefore I'm stuck.
    – Fluffy Skye
    4 hours ago












up vote
1
down vote

favorite









up vote
1
down vote

favorite











Let $a_n$ be a sequence of real non-negative numbers



Define $S_n = fracsum_i=1^n a_in$



Prove that



$$liminf(a_n) leq liminf(S_n) leq limsup(S_n) leq limsup(a_n)$$



I wanted to show that $infa_n:n geq t leq infS_n:ngeq t$ for all $t$



However this doesn't seem to be true



The inequality seems to hold only at n goes to infinity but not every n individually



Can anyone help me?










share|cite|improve this question















Let $a_n$ be a sequence of real non-negative numbers



Define $S_n = fracsum_i=1^n a_in$



Prove that



$$liminf(a_n) leq liminf(S_n) leq limsup(S_n) leq limsup(a_n)$$



I wanted to show that $infa_n:n geq t leq infS_n:ngeq t$ for all $t$



However this doesn't seem to be true



The inequality seems to hold only at n goes to infinity but not every n individually



Can anyone help me?







real-analysis limsup-and-liminf






share|cite|improve this question















share|cite|improve this question













share|cite|improve this question




share|cite|improve this question








edited 4 hours ago









GNUSupporter 8964民主女神 地下教會

12.1k72243




12.1k72243










asked 4 hours ago









Fluffy Skye

688




688











  • To see why your proposed inequality is false, you may take the first term to be strictly smaller than the rest of the sequence.
    – GNUSupporter 8964民主女神 地下教會
    4 hours ago










  • @GNUSupporter 8964民主女神 地下教會 Yes that is what I found and therefore I'm stuck.
    – Fluffy Skye
    4 hours ago
















  • To see why your proposed inequality is false, you may take the first term to be strictly smaller than the rest of the sequence.
    – GNUSupporter 8964民主女神 地下教會
    4 hours ago










  • @GNUSupporter 8964民主女神 地下教會 Yes that is what I found and therefore I'm stuck.
    – Fluffy Skye
    4 hours ago















To see why your proposed inequality is false, you may take the first term to be strictly smaller than the rest of the sequence.
– GNUSupporter 8964民主女神 地下教會
4 hours ago




To see why your proposed inequality is false, you may take the first term to be strictly smaller than the rest of the sequence.
– GNUSupporter 8964民主女神 地下教會
4 hours ago












@GNUSupporter 8964民主女神 地下教會 Yes that is what I found and therefore I'm stuck.
– Fluffy Skye
4 hours ago




@GNUSupporter 8964民主女神 地下教會 Yes that is what I found and therefore I'm stuck.
– Fluffy Skye
4 hours ago










2 Answers
2






active

oldest

votes

















up vote
3
down vote



accepted










The statement follows directly by Stolz–Cesàro theorem



$$liminf fracA_n+1-A_nB_n+1-B_n leq liminf fracA_nB_n leq limsup fracA_nB_n leq limsup fracA_n+1-A_nB_n+1-B_n$$



by $A_n=S_n$ and $B_n=n$.






share|cite|improve this answer






















  • I am new to real analysis and haven't learnt this theorem yet. Do you have any more elementary method?
    – Fluffy Skye
    4 hours ago










  • This gave me some trouble, until I realised that the $a_n$ from the theorem is supposed to be $sum_k=1^n a_k$, not $a_n$.
    – Theo Bendit
    4 hours ago










  • Also you meant $a_n = S_n$ Right?
    – Fluffy Skye
    4 hours ago










  • @TheoBendit Opsss yes of course I fix that!
    – gimusi
    4 hours ago










  • @FluffySkye A way is to prove Stolz-Cesaro theorem and then use that.
    – gimusi
    4 hours ago

















up vote
2
down vote













Let $l$ be the $liminf$ of $a_n$. Then for all $varepsilon > 0$, there exists an $N in mathbbN$ such that
$$n ge N implies a_n ge l - varepsilon.$$
Let $M = sum_k=1^N-1 a_k.$ Then,
$$n ge N implies sum_k=1^n a_n = M + sum_k=N^n a_n ge M + (n - N + 1)(l - varepsilon).$$
Therefore, when $n ge N$, we have
$$fracsum_k=1^n a_nn ge fracMn + left(1 - fracN - 1nright)(l - varepsilon) to l - varepsilon.$$
Hence, every subsequence of $fracsum_k=1^n a_nn$ must converge to at least $l - varepsilon$, where $varepsilon > 0$ is arbitrary. Thus, every subsequence must converge to at least $l$, proving the inequality of the limits inferior.






share|cite|improve this answer




















    Your Answer





    StackExchange.ifUsing("editor", function ()
    return StackExchange.using("mathjaxEditing", function ()
    StackExchange.MarkdownEditor.creationCallbacks.add(function (editor, postfix)
    StackExchange.mathjaxEditing.prepareWmdForMathJax(editor, postfix, [["$", "$"], ["\\(","\\)"]]);
    );
    );
    , "mathjax-editing");

    StackExchange.ready(function()
    var channelOptions =
    tags: "".split(" "),
    id: "69"
    ;
    initTagRenderer("".split(" "), "".split(" "), channelOptions);

    StackExchange.using("externalEditor", function()
    // Have to fire editor after snippets, if snippets enabled
    if (StackExchange.settings.snippets.snippetsEnabled)
    StackExchange.using("snippets", function()
    createEditor();
    );

    else
    createEditor();

    );

    function createEditor()
    StackExchange.prepareEditor(
    heartbeatType: 'answer',
    convertImagesToLinks: true,
    noModals: true,
    showLowRepImageUploadWarning: true,
    reputationToPostImages: 10,
    bindNavPrevention: true,
    postfix: "",
    imageUploader:
    brandingHtml: "Powered by u003ca class="icon-imgur-white" href="https://imgur.com/"u003eu003c/au003e",
    contentPolicyHtml: "User contributions licensed under u003ca href="https://creativecommons.org/licenses/by-sa/3.0/"u003ecc by-sa 3.0 with attribution requiredu003c/au003e u003ca href="https://stackoverflow.com/legal/content-policy"u003e(content policy)u003c/au003e",
    allowUrls: true
    ,
    noCode: true, onDemand: true,
    discardSelector: ".discard-answer"
    ,immediatelyShowMarkdownHelp:true
    );



    );













     

    draft saved


    draft discarded


















    StackExchange.ready(
    function ()
    StackExchange.openid.initPostLogin('.new-post-login', 'https%3a%2f%2fmath.stackexchange.com%2fquestions%2f2981070%2fa-question-about-limsup-of-the-sequence-of-the-average%23new-answer', 'question_page');

    );

    Post as a guest






























    2 Answers
    2






    active

    oldest

    votes








    2 Answers
    2






    active

    oldest

    votes









    active

    oldest

    votes






    active

    oldest

    votes








    up vote
    3
    down vote



    accepted










    The statement follows directly by Stolz–Cesàro theorem



    $$liminf fracA_n+1-A_nB_n+1-B_n leq liminf fracA_nB_n leq limsup fracA_nB_n leq limsup fracA_n+1-A_nB_n+1-B_n$$



    by $A_n=S_n$ and $B_n=n$.






    share|cite|improve this answer






















    • I am new to real analysis and haven't learnt this theorem yet. Do you have any more elementary method?
      – Fluffy Skye
      4 hours ago










    • This gave me some trouble, until I realised that the $a_n$ from the theorem is supposed to be $sum_k=1^n a_k$, not $a_n$.
      – Theo Bendit
      4 hours ago










    • Also you meant $a_n = S_n$ Right?
      – Fluffy Skye
      4 hours ago










    • @TheoBendit Opsss yes of course I fix that!
      – gimusi
      4 hours ago










    • @FluffySkye A way is to prove Stolz-Cesaro theorem and then use that.
      – gimusi
      4 hours ago














    up vote
    3
    down vote



    accepted










    The statement follows directly by Stolz–Cesàro theorem



    $$liminf fracA_n+1-A_nB_n+1-B_n leq liminf fracA_nB_n leq limsup fracA_nB_n leq limsup fracA_n+1-A_nB_n+1-B_n$$



    by $A_n=S_n$ and $B_n=n$.






    share|cite|improve this answer






















    • I am new to real analysis and haven't learnt this theorem yet. Do you have any more elementary method?
      – Fluffy Skye
      4 hours ago










    • This gave me some trouble, until I realised that the $a_n$ from the theorem is supposed to be $sum_k=1^n a_k$, not $a_n$.
      – Theo Bendit
      4 hours ago










    • Also you meant $a_n = S_n$ Right?
      – Fluffy Skye
      4 hours ago










    • @TheoBendit Opsss yes of course I fix that!
      – gimusi
      4 hours ago










    • @FluffySkye A way is to prove Stolz-Cesaro theorem and then use that.
      – gimusi
      4 hours ago












    up vote
    3
    down vote



    accepted







    up vote
    3
    down vote



    accepted






    The statement follows directly by Stolz–Cesàro theorem



    $$liminf fracA_n+1-A_nB_n+1-B_n leq liminf fracA_nB_n leq limsup fracA_nB_n leq limsup fracA_n+1-A_nB_n+1-B_n$$



    by $A_n=S_n$ and $B_n=n$.






    share|cite|improve this answer














    The statement follows directly by Stolz–Cesàro theorem



    $$liminf fracA_n+1-A_nB_n+1-B_n leq liminf fracA_nB_n leq limsup fracA_nB_n leq limsup fracA_n+1-A_nB_n+1-B_n$$



    by $A_n=S_n$ and $B_n=n$.







    share|cite|improve this answer














    share|cite|improve this answer



    share|cite|improve this answer








    edited 4 hours ago

























    answered 4 hours ago









    gimusi

    80.8k74090




    80.8k74090











    • I am new to real analysis and haven't learnt this theorem yet. Do you have any more elementary method?
      – Fluffy Skye
      4 hours ago










    • This gave me some trouble, until I realised that the $a_n$ from the theorem is supposed to be $sum_k=1^n a_k$, not $a_n$.
      – Theo Bendit
      4 hours ago










    • Also you meant $a_n = S_n$ Right?
      – Fluffy Skye
      4 hours ago










    • @TheoBendit Opsss yes of course I fix that!
      – gimusi
      4 hours ago










    • @FluffySkye A way is to prove Stolz-Cesaro theorem and then use that.
      – gimusi
      4 hours ago
















    • I am new to real analysis and haven't learnt this theorem yet. Do you have any more elementary method?
      – Fluffy Skye
      4 hours ago










    • This gave me some trouble, until I realised that the $a_n$ from the theorem is supposed to be $sum_k=1^n a_k$, not $a_n$.
      – Theo Bendit
      4 hours ago










    • Also you meant $a_n = S_n$ Right?
      – Fluffy Skye
      4 hours ago










    • @TheoBendit Opsss yes of course I fix that!
      – gimusi
      4 hours ago










    • @FluffySkye A way is to prove Stolz-Cesaro theorem and then use that.
      – gimusi
      4 hours ago















    I am new to real analysis and haven't learnt this theorem yet. Do you have any more elementary method?
    – Fluffy Skye
    4 hours ago




    I am new to real analysis and haven't learnt this theorem yet. Do you have any more elementary method?
    – Fluffy Skye
    4 hours ago












    This gave me some trouble, until I realised that the $a_n$ from the theorem is supposed to be $sum_k=1^n a_k$, not $a_n$.
    – Theo Bendit
    4 hours ago




    This gave me some trouble, until I realised that the $a_n$ from the theorem is supposed to be $sum_k=1^n a_k$, not $a_n$.
    – Theo Bendit
    4 hours ago












    Also you meant $a_n = S_n$ Right?
    – Fluffy Skye
    4 hours ago




    Also you meant $a_n = S_n$ Right?
    – Fluffy Skye
    4 hours ago












    @TheoBendit Opsss yes of course I fix that!
    – gimusi
    4 hours ago




    @TheoBendit Opsss yes of course I fix that!
    – gimusi
    4 hours ago












    @FluffySkye A way is to prove Stolz-Cesaro theorem and then use that.
    – gimusi
    4 hours ago




    @FluffySkye A way is to prove Stolz-Cesaro theorem and then use that.
    – gimusi
    4 hours ago










    up vote
    2
    down vote













    Let $l$ be the $liminf$ of $a_n$. Then for all $varepsilon > 0$, there exists an $N in mathbbN$ such that
    $$n ge N implies a_n ge l - varepsilon.$$
    Let $M = sum_k=1^N-1 a_k.$ Then,
    $$n ge N implies sum_k=1^n a_n = M + sum_k=N^n a_n ge M + (n - N + 1)(l - varepsilon).$$
    Therefore, when $n ge N$, we have
    $$fracsum_k=1^n a_nn ge fracMn + left(1 - fracN - 1nright)(l - varepsilon) to l - varepsilon.$$
    Hence, every subsequence of $fracsum_k=1^n a_nn$ must converge to at least $l - varepsilon$, where $varepsilon > 0$ is arbitrary. Thus, every subsequence must converge to at least $l$, proving the inequality of the limits inferior.






    share|cite|improve this answer
























      up vote
      2
      down vote













      Let $l$ be the $liminf$ of $a_n$. Then for all $varepsilon > 0$, there exists an $N in mathbbN$ such that
      $$n ge N implies a_n ge l - varepsilon.$$
      Let $M = sum_k=1^N-1 a_k.$ Then,
      $$n ge N implies sum_k=1^n a_n = M + sum_k=N^n a_n ge M + (n - N + 1)(l - varepsilon).$$
      Therefore, when $n ge N$, we have
      $$fracsum_k=1^n a_nn ge fracMn + left(1 - fracN - 1nright)(l - varepsilon) to l - varepsilon.$$
      Hence, every subsequence of $fracsum_k=1^n a_nn$ must converge to at least $l - varepsilon$, where $varepsilon > 0$ is arbitrary. Thus, every subsequence must converge to at least $l$, proving the inequality of the limits inferior.






      share|cite|improve this answer






















        up vote
        2
        down vote










        up vote
        2
        down vote









        Let $l$ be the $liminf$ of $a_n$. Then for all $varepsilon > 0$, there exists an $N in mathbbN$ such that
        $$n ge N implies a_n ge l - varepsilon.$$
        Let $M = sum_k=1^N-1 a_k.$ Then,
        $$n ge N implies sum_k=1^n a_n = M + sum_k=N^n a_n ge M + (n - N + 1)(l - varepsilon).$$
        Therefore, when $n ge N$, we have
        $$fracsum_k=1^n a_nn ge fracMn + left(1 - fracN - 1nright)(l - varepsilon) to l - varepsilon.$$
        Hence, every subsequence of $fracsum_k=1^n a_nn$ must converge to at least $l - varepsilon$, where $varepsilon > 0$ is arbitrary. Thus, every subsequence must converge to at least $l$, proving the inequality of the limits inferior.






        share|cite|improve this answer












        Let $l$ be the $liminf$ of $a_n$. Then for all $varepsilon > 0$, there exists an $N in mathbbN$ such that
        $$n ge N implies a_n ge l - varepsilon.$$
        Let $M = sum_k=1^N-1 a_k.$ Then,
        $$n ge N implies sum_k=1^n a_n = M + sum_k=N^n a_n ge M + (n - N + 1)(l - varepsilon).$$
        Therefore, when $n ge N$, we have
        $$fracsum_k=1^n a_nn ge fracMn + left(1 - fracN - 1nright)(l - varepsilon) to l - varepsilon.$$
        Hence, every subsequence of $fracsum_k=1^n a_nn$ must converge to at least $l - varepsilon$, where $varepsilon > 0$ is arbitrary. Thus, every subsequence must converge to at least $l$, proving the inequality of the limits inferior.







        share|cite|improve this answer












        share|cite|improve this answer



        share|cite|improve this answer










        answered 4 hours ago









        Theo Bendit

        15.6k12147




        15.6k12147



























             

            draft saved


            draft discarded















































             


            draft saved


            draft discarded














            StackExchange.ready(
            function ()
            StackExchange.openid.initPostLogin('.new-post-login', 'https%3a%2f%2fmath.stackexchange.com%2fquestions%2f2981070%2fa-question-about-limsup-of-the-sequence-of-the-average%23new-answer', 'question_page');

            );

            Post as a guest













































































            Comments

            Popular posts from this blog

            Long meetings (6-7 hours a day): Being “babysat” by supervisor

            Is the Concept of Multiple Fantasy Races Scientifically Flawed? [closed]

            Confectionery